ein Beispiel, bei dem die Änderung des Bezugsrahmens eines Beobachters den Ausgang von Ereignissen verändert!

Betrachten Sie zwei identische Ladungen, die sich mit gleichförmiger Geschwindigkeit bewegen. Zwischen ihnen besteht eine magnetische Anziehungskraft, da sich zwei Ströme in derselben Richtung anziehen. Wenn ich auf einer der Ladungen sitze, bewegt sich meiner Meinung nach die andere Ladung nicht. Es wird also keine magnetische Anziehungskraft geben. Wie verändert eine Änderung des Bezugsrahmens das Ergebnis der Interaktion? Was wird eigentlich passieren?

Zwischen den beiden gleichnamigen Ladungen besteht zunächst eine elektrostatische Abstoßungskraft,
F = Q 1 Q 2 4 π ϵ 0 R 2
Das ist es in dem Rahmen, in dem die Ladungen ruhen. In einem Rahmen, der sich durch Geschwindigkeit bewegt v senkrecht zur Trennung der Ladungen schauen beide Ladungen γ mal schwerer, weil sie sich bewegen. Die gleiche elektrostatische Kraft wird daher eine geringere Beschleunigung verursachen, was in Ordnung ist - es kann als Zeitdilatation interpretiert werden. Es wird auch ein Magnetfeld geben v × E / C 2 in diesem beweglichen Rahmen.
Dieses zusätzliche Magnetfeld B wird in der Tat eine positive Anziehungskraft beitragen, die Sie berechnen können - es wird die ursprüngliche Abstoßungskraft noch weiter reduzieren. Wenn Sie all diese Dinge kombinieren, werden Sie sehen, dass die Ladungen wie erwartet von einer Transformation der Trajektorien beschleunigt werden.
Besuchen Sie die Seite: Klassischer Elektromagnetismus und spezielle Relativitätstheorie . Lesen Sie, bis Sie die Worte "Dies bedeutet nicht, dass zwei völlig unterschiedliche Ereignisreihen in den beiden Bildern zu sehen sind, sondern dass dieselbe Ereignisfolge auf zwei verschiedene Arten beschrieben wird" finden, Ihre Frage wird beantwortet.
Nützliche Links: Beitrag in der Physik SE mit zwei netten Antworten zu Ihrer Frage: physical.stackexchange.com/questions/38151/… Und ein Link zu Feynmans Band II, Kapitel 13-6, der die Relativität von Magnetfeldern behandelt (kostenlos gemacht). online zu lesen bei Caltech) feynmanlectures.caltech.edu/II_13.html#Ch13-S6

Antworten (2)

Sie haben Recht; in dem Rahmen, der sich mit den Ladungen bewegt, gibt es keine magnetische Anziehung. Es wird jedoch eine elektrostatische Abstoßung geben, sodass Sie sehen werden, wie sich die beiden Partikel voneinander entfernen.

Stellen Sie sich ein anderes Bild (das Ruhebild) vor, in dem sich beide Partikel nebeneinander in x-Richtung bewegen (ich nehme an, das ist das, was Sie sich vorstellen). Noch stoßen sich die Teilchen elektrostatisch ab, aber jetzt ziehen sie sich auch noch magnetisch an!

Dies mag wie ein Problem erscheinen, ist es aber nicht, denn obwohl die im Ruhesystem gemessene Kraft schwächer ist, da die elektrostatische Abstoßung teilweise durch die magnetische Anziehung aufgehoben wird, ist die Zeit, die von sich bewegenden Beobachtern erlebt wird, aufgrund der Hexerei von ebenfalls unterschiedlich Spezielle Relativität. Wenn der Rahmen, der sich mit den Ladungen bewegt, grundiert ist und der Ruherahmen nicht grundiert ist, dann

T ' = γ T F = Q ( E + v × B ) = Q ( E v B ) F ' = Q E '

Was wir zeigen wollen ist, dass, da die Kraft umgekehrt proportional zum Quadrat der Zeit ist, F und F' durch einen Faktor von in Beziehung stehen γ 2 in diesem Fall berechnen Beobachter in beiden Rahmen übereinstimmende Trajektorien unter Verwendung der Maxwell-Gleichungen.

Da für eine Punktgebühr

E = 1 4 π ϵ Q R 2

B = μ 4 π Q v × R ^ R 2 = μ 4 π Q v R 2

Jetzt haben wir alles, was wir brauchen. B an F anschließen und verwenden μ = 1 / ( C 2 ϵ ) Und γ = 1 / 1 v 2 C 2

F = Q ( E v B ) = Q ( E v 2 C 2 1 4 π ϵ Q R 2 ) = Q γ 2 E

F ' = Q E ' = Q E = 1 γ 2 F

Was wir seitdem die ganze Zeit wollten

F = D 2 j ' D T ' 2 = D 2 j ( γ   D T ) 2 = 1 γ 2 F

(wobei zu beachten ist, dass dy von einem Boost in x-Richtung nicht beeinflusst wird)

Um kompliziertere Probleme dieser Art zu lösen, möchten Sie normalerweise über Faraday-Tensoren, 4-Ströme und Einstein-Notation sprechen, aber ich hoffe, dieses einfache Beispiel hat Ihnen einen Einblick gegeben, wie Elektromagnetismus und spezielle Relativitätstheorie zusammenhängen. Sie haben sich vielleicht gefragt, was passiert, wenn sich die Teilchen so schnell bewegen, dass die magnetische Kraft die elektrostatische Abstoßung übersteigt, aber Sie können tatsächlich sehen, dass Sie dazu die Lichtgeschwindigkeit überschreiten müssen!

Wenn Sie verstehen möchten, warum der Elektromagnetismus offensichtlich robust gegen Relativitätsparadoxien ist, sollten Sie sich über die Tensoranalyse in der 4D-Raumzeit informieren, die für jeden Physiker, der sein Geld wert ist, ein Muss ist und hinter den meisten Mechanikern zu finden ist oder Elektrodynamik-Lehrbücher oder am Anfang der meisten allgemeinen Relativitätstheorie-Bücher. Wenn Sie eine körperliche Intuition dafür gewinnen möchten, wie EM funktioniert, empfehle ich Purcell vereinfacht.

http://physics.weber.edu/schroeder/mrr/MRRtalk.html

In Feynamn-Vorlesungen zeigt er am Beispiel einer Ladung, die sich parallel zu einem Draht bewegt, ziemlich direkt, dass eine vollständige elektromagnetische Beschreibung invariant zum Trägheitsbezugssystem ist, dh Elektrizität und Magnetismus zusammengenommen sind konsistent mit Einsteins Relativität.

In Fällen wie Ihrem Beispiel müssen Sie also immer im Hinterkopf behalten, wie die Mischung der Elektrik ist E und magnetisch B Felder ändert sich von einem Beobachter zum anderen. Einige Zitate aus seinem Buch:

Es stellt sich heraus, dass jeder Trägheitsrahmen ausreicht. Wir werden auch sehen, dass Magnetismus und Elektrizität keine unabhängigen Dinge sind – dass sie immer als ein vollständiges elektromagnetisches Feld betrachtet werden sollten. Obwohl sich die Maxwellschen Gleichungen im statischen Fall in zwei unterschiedliche Paare trennen, ein Paar für Elektrizität und ein Paar für Magnetismus, ohne offensichtliche Verbindung zwischen den beiden Feldern, besteht dennoch in der Natur selbst eine sehr enge Beziehung zwischen ihnen, die sich aus dem Prinzip ergibt der Relativität. Historisch gesehen wurde das Relativitätsprinzip nach den Maxwellschen Gleichungen entdeckt. Tatsächlich war es das Studium der Elektrizität und des Magnetismus, das letztendlich zu Einsteins Entdeckung seines Relativitätsprinzips führte.

Ein Beispiel für relativistische Faktoren, die unter Verwendung der Gesetze des Elektromagnetismus vor der speziellen Relativitätstheorie ins Spiel kommen, finden Sie hier , wenn Sie interessiert sind.

Weiter in seiner Herleitung werden Sie sehen, dass Ladungsdichten auch rahmenabhängig sind, damit das Gesamtladungserhaltungsgesetz gilt. Sauber!

Um zum Endergebnis zu kommen:

Wir haben festgestellt, dass wir das gleiche physikalische Ergebnis erhalten, egal ob wir die Bewegung eines Teilchens analysieren, das sich entlang eines Drahtes in einem bezüglich des Drahtes ruhenden Koordinatensystem bewegt, oder in einem bezüglich des Teilchens ruhenden System. Im ersten Fall war die Kraft rein „magnetisch“, im zweiten rein „elektrisch“.

Ich habe mit Feynmans Beispiel begonnen, nur um etwas Kontext zu setzen, jetzt zurück zu unserem Fall, ohne Draht, wie Luboš in Kommentaren betonte, im Bezugsrahmen der sich bewegenden Ladungen gibt es nur die Coulomb-Abstoßung zwischen den beiden Protonen (oder zwei Elektronen), während im Rahmen des Labors (bewegte Ladungen) die Coulomb-Abstoßung mit einer magnetischen Kraft gekoppelt ist, die die beiden anzieht. Gemäß der zuvor dargelegten Argumentation wird der elektrische und magnetische Feldübergang zwischen den beiden erwähnten Rahmen einfach erhalten, indem die Lorentz-Transformationen auf die Felder eines Rahmens angewendet werden, um den anderen zu erhalten. Hier können wir für unser spezielles Problem einfach mit den transversalen Komponenten arbeiten E , bezeichnet als E T .

Im übrigen Gebührenrahmen S , haben wir nur ein elektrisches Feld, dessen Traversalkomponente bezeichnet ist E T , die Lorentz-Transformation gibt das transversale elektrische Feld an S ' Ist:

E T ' = γ ( E T | v × B | )

Beachten Sie das Auftreten eines Magnetfelds im Ausdruck!

Nachdem wir nun die Felder in beiden Frames definiert haben, wissen wir, dass die Ladungen in jedem Frame EM-Kräften ausgesetzt sind. Um die Physik zu überprüfen, lassen Sie uns also herausfinden, welche transversalen Impulse jeder Ladung haben, nachdem die EM-Kraft gewirkt hat auf ihnen für eine bestimmte Zeit. Wir erwarten, dass die Ergebnisse in beiden Frames gleich sind.

Verwendung der richtigen relativistischen Bewegungsgleichung F = D P / D T , nach dem Zeitintervall Δ T wir erwarten, dass die Querimpulsänderung sein wird Δ P T = F Δ T In S , Und Δ P T ' = F ' Δ T ' In S ' . Das gleiche Ergebnis der Querimpulsänderung in beiden Rahmen zu erhalten, bedeutet, dass die folgende Beziehung gelten sollte:

Δ P T Δ P T ' = 1 = F Δ T F ' Δ T '

Wo für ein beliebiges Zeitintervall Δ T In S , das entsprechende Zeitintervall in S ' wird dilatiert (Zeitdilatation in bewegten Bildern):

Δ T ' = γ Δ T

Jetzt müssen wir nur noch den Ausdruck von finden F ' , sollte das erwartete Ergebnis den Effekt der Zeitdilatation aufheben, damit der Impuls in Querrichtung erhalten bleibt. Das wissen wir im S ' Jede Ladung ist auch einem Magnetfeld ausgesetzt, und seine Wirkung sollte die Coulomb-Abstoßung verringern. Wissen E T ' (von früher) haben wir die gesamte EM-Kraft auf jede Ladung S ' wird gegeben von:

F ' = γ Q ( E T v B ) = γ Q ( E T v ( v E T C 2 ) ) F ' = γ Q E T ( 1 v 2 C 2 )

Ersetzen Δ T ' Und F T ' durch die obigen Ausdrücke erhalten wir:

F Δ T F ' Δ T ' = F Δ T γ 2 ( 1 v 2 C 2 ) F Δ T = 1
Was ja gleich 1 ist, da γ = ( 1 v 2 / C 2 ) 1 / 2 .

Daher wird das gleiche physikalische Ergebnis von beiden Bezugsrahmen erhalten, S Und S ' , kein Paradoxon! Zum Abschluss noch einmal ein Zitat von Feynman:

Eine vollständige elektromagnetische Beschreibung ist unveränderlich; Elektrizität und Magnetismus zusammengenommen stimmen mit Einsteins Relativitätstheorie überein.

Weitere nützliche Lektüre: Feynman Lectures, Vol II Kapitel 13-6.

Lorentztransformation der Felder

Lorentz-Transformation für EM, aus Wikipedia.